LSAT and Law School Admissions Forum

Get expert LSAT preparation and law school admissions advice from PowerScore Test Preparation.

 sarae
  • Posts: 80
  • Joined: Aug 10, 2013
|
#10861
Why is E correct vs. B?
 Steve Stein
PowerScore Staff
  • PowerScore Staff
  • Posts: 1153
  • Joined: Apr 11, 2011
|
#10874
Hi sarae,

In that one, based on the fact that a reduction in speed limit was followed by a reduction in highway fatalities, the author concludes that reducing the speed limit can cause a reduction in traffic fatalities.

The question asks what the author takes for granted. As provided by answer choice E. The author is presuming that the noted fatalities decrease was not explained by an abnormally high number the year before.

Answer choice B claims that the author takes for granted that most drivers obeyed the speed limit, but the author does not claim or imply that the speed limit was obeyed by the majority, just that the reduced speed limit led to the reduced number of fatailities.

I hope that's helpful! Please let me know whether this is clear--thanks!

~Steve
 sarae
  • Posts: 80
  • Joined: Aug 10, 2013
|
#10901
got it!
 nutcracker
  • Posts: 39
  • Joined: Aug 13, 2017
|
#43210
Hello! I need some help with this question.

In my understanding, the question is asking for an assumption of the argument. When I use the assumption negation technique, however, both (C) and (E) seem to weaken the argument then negated, which I think is not supposed to happen. What went wrong in my solution? How do I choose one over the other?

In addition, I would also appreciate an analysis of answer choice (B). Thanks a lot!
 Claire Horan
PowerScore Staff
  • PowerScore Staff
  • Posts: 408
  • Joined: Apr 18, 2016
|
#43220
I'm glad to hear you are using the Assumption Negation technique!

Step Zero, if you will, of the Assumption Negation technique, though, is that the statement you are negating has to actually be an assumption made by the argument. The stem asks you for something the argument takes for granted, so you can't just choose any random statement that, when negated, would attack the conclusion. It has to be an assumption that was made by the argument. This is why answer C fails. The argument doesn't assume there is a relation, it concludes there is a (causal) relation. (Remember, an assumption is an unstated premise, not a conclusion.)

E is an assumption made in the argument because if the year previous was an outlier, we wouldn't be able to draw any conclusions from the next year's return to relative normal.

Answer choice B goes much too far--it is not necessary for the conclusion to be drawn. What if none of the drivers obey the speed limit, but they do slow down somewhat? The conclusion is not attacked.

The takeaway: Assumptions are unstated premises. If a statement is made explicitly in the argument, it is not an assumption.
 akanshalsat
  • Posts: 104
  • Joined: Dec 20, 2017
|
#48754
If this is a causation-correlation question - then isnt C correct in that they are saying that speed is causing the lesser amounts of fatalities, thus saying there is the relation between speed and accidents?

E is still confusing for me, if someone could explain exactly how it reveals the "flaw" in the argument.

I guess i'm getting mixed up bc this is supposedly a flaw question but we're supposed to see it in terms of an assumption question??

Super confused :(
 Adam Tyson
PowerScore Staff
  • PowerScore Staff
  • Posts: 5153
  • Joined: Apr 14, 2011
|
#48775
There is a close relationship between Flaw and Assumption question, akanshalsat, in that pretty much every flaw can be described as a bad assumption. For example, in a source argument, where you attack the person who made the argument instead of the merits of the argument itself, you can see that as the author assuming that a "bad person" can't make a good argument. In a Time Shift flaw, the author improperly assumes that what happened in the past will happen in the future. So while we are looking for the flaw in the reasoning here, that is very much like looking for their unwarranted or questionable assumption.

Now let's take a look at answer C. Why is this not the flaw in the argument, the author's unwarranted assumption? Two reasons - first, he made no argument about actual driving speed, but only about speed limits. A fine point, but an important difference! Second, he made no argument about fewer accidents, only fewer fatalities! Maybe he didn't assume fewer accidents, only ones that are less severe?

Now think about answer E in terms of causal reasoning. The author is claiming that the lower speed limit caused the lower number of fatalities. To make that claim, he has to assume that there was no other cause for the lower number of fatalities this year. That means that, among other things, he assumed that the change wasn't due to a return to normal. Maybe the speed limit had nothing to do with it, and the change was due to last year being anomalous? That could have "caused" the change, instead of the speed limit reduction, right? Maybe instead of looking at what helped this year, we should be looking to find out what made last year so unusual? The author assumes that it wasn't unusually high, and that assumption may not be valid.

I hope that helps!
User avatar
 sunshine123
  • Posts: 44
  • Joined: Jul 18, 2022
|
#97945
Hello,

There seem to a number of reasons floating around as to why C is wrong; one proctors says, "This is why answer C fails. The argument doesn't assume there is a relation, it concludes there is a (causal) relation. (Remember, an assumption is an unstated premise, not a conclusion." Another proctors points out that the author may have assumed that there is a relation between speed limit and driving accidents, but not necessarily a relation between driving speed and driving accidents. I personally find the latter explanation more satisfactory, while the former one left me deeply confused -- you can't derive a causal conclusion from correlative evidence unless you make some causal assumptions. Clarification would be greatly appreciated. Thanks
 Robert Carroll
PowerScore Staff
  • PowerScore Staff
  • Posts: 1783
  • Joined: Dec 06, 2013
|
#98279
sunshine123,

The problem with answer choice (C) is definitely the latter - the argument is not about number of accidents but number of fatalities, and not about actual driving speed but speed limits.

Robert Carroll
User avatar
 SiyaMajmundar
  • Posts: 1
  • Joined: Feb 28, 2024
|
#105464
Hello everyone!

B is incorrect for 1 major reason. But first, let’s determine the flaw and go through the answer choices. The argument states that the reduction in highway speed led to the reduction [can reduce] of traffic fatalities. The flaw here is that the argument is assuming that nothing else changed when the highway speed limits were enacted. For example, it wasn’t the case less people started driving on the highway, or that automobiles were now equipped with softwares that prevented accidents, etc. So here, we have the first step done, which is determining the flaw. But the question is not asking us to identify the flaw - it’s asking us to determine what the argument is assuming as a result of the flaw. So think Necessary Assumption. If it were the case that the speed limits in fact led to the reduction in traffic fatalities what must we assume?

A- We don’t need to assume this. Even is it has so what? What is this doing?

B- The majority of the drivers obeyed the traffic laws. Yes this is good as a Sufficient Assumption, but is it a Necessary one? Is it necessary for traffic laws to be obeyed in order for speed limits to be enacted. What if it was the case that the enaction of the speed limit deterred people from using the highway, and now there are 10 cars on the highway the entire day who are all not following the speed limit? See, we don’t need to assume that the speed limits had to be followed.

C- The argument is not assuming this, it explicitly stated this.

D- So what if it was strictly enforced than the old? It could have been less strictly enforced than the old and still provided these results.

E- Yes, this is a NA that comes with this flawed argument. If it were the case that normally traffic fatalities average 20/year and the previous year they had an outlier number, for example 190/year, and this year it went to 20, this gives a reason to question the stimulus.

I hope this helped!

Get the most out of your LSAT Prep Plus subscription.

Analyze and track your performance with our Testing and Analytics Package.